Purchase Solution

Eigenfunctions - Expand the function

Not what you're looking for?

Ask Custom Question

Please show each step of your solution. When you use theorems, definitions, etc., please include in your answer.

Expand the function

f(x) = {x^4, 0 <= x < 2
{0, 2 <= x <= pi

in terms of the eigenfunctions of the given eigenvalue problem. Use computer software, such as the Maple int command, to evaluate the expansion coefficient a_n as a function of n.

(a) y" + lambda y = 0, y'(0) = 0, y'(pi) = 0
(b) y" + lambda y = 0, y'(0) = 0, y(pi) = 0
(b) y" + lambda y = 0, y(0) = 0, y'(pi) = 0

Please see the attached.

Attachments
Purchase this Solution

Solution Summary

This shows how to expand a given function in terms of eigenfunctions of a given eigenvalue problem. B is solved in this solution, which is step by step and detailed, including diagrams.

Solution Preview

Solution:

We start with solving the Sturm-Liouville (S-L) problem (b), whose general solution is:

See attached

The boundary conditions lead to:

See attached

Since A couldn’t be zero, we get an eigenvalues problem, that is there are certain values of (λ) which satisfy (4):

See attached

These are the eigenvalues of our S-L problem. The ...

Purchase this Solution


Free BrainMass Quizzes
Exponential Expressions

In this quiz, you will have a chance to practice basic terminology of exponential expressions and how to evaluate them.

Graphs and Functions

This quiz helps you easily identify a function and test your understanding of ranges, domains , function inverses and transformations.

Solving quadratic inequalities

This quiz test you on how well you are familiar with solving quadratic inequalities.

Multiplying Complex Numbers

This is a short quiz to check your understanding of multiplication of complex numbers in rectangular form.

Know Your Linear Equations

Each question is a choice-summary multiple choice question that will present you with a linear equation and then make 4 statements about that equation. You must determine which of the 4 statements are true (if any) in regards to the equation.